Howdy, Stranger!

It looks like you're new here. If you want to get involved, click one of these buttons!

PT18 S4 Q19

marco.delgmarco.delg Alum Member
edited July 2020 in Logical Reasoning 44 karma

help

I answered this question correctly but approached the stimulus differently than J.Y. I interpreted the part that says “who receive unsolicited advice from someone whose advantage would be served if that advice is taken” to be a description of the kind of “people” the stimulus is referring to. Therefore, I teased out the logic in this way:

People (who receive unsolicited advice from someone whose advantage would be served if that advice is taken)

Interest NOT coincide → Regard advice with Skepticism

so

NOT IC → SK

Therefore, I approached the answer choices in this way:

A – Even if the interest of H and F do not coincide, the stimulus does not suggest a rejection;

B – Perfectly resembled my interpretation since S and R “NOT IC” then R should not reject the least expensive models. So, he must be skeptical about S’s advice;

C – In this case M and Y interest coincided so it did not trigger my conditional;

D – In this case S and R interest coincided so it did not trigger my conditional;

E – Even if we can infer that M and J interest did not coincide because M wanted to purchase a more expensive fish while J wanted to sell her a cheaper one, in no way we can conclude that M should follow J’s recommendation, so “NOT SK”.

What do you think?

Thank you in advance to whoever is going to answer.

Best,

Marco

Admin Note: https://7sage.com/lsat_explanations/lsat-18-section-4-question-19/

Sign In or Register to comment.